[obm-l] Problema!

2007-08-31 Thread jose silva


Um grupo de trabalhadores tinha a tarefa de realizar a colheita de dois 
campos de trigo, um dos quais tinha o dobro da área do outro.

Durante meio dia todo o pessoal do grupo trabalhou no campo de trigo grande.
Depois do almoço, metade do pessoal continuou no campo de trigo grande e a 
outra metade trabalhou no campo de trigo pequeno.
Durante a tarde acabaram ambos as tarefas exceto uma pequena área  do campo 
de trigo menor, em cuja colheita se ocupou todo o dia seguinte apenas um 
trabalhador.

Quantos trabalhadores haviam neste grupo?

Claudio.

_
Verificador de Segurança do Windows Live OneCare: verifique já a segurança 
do seu PC! http://onecare.live.com/site/pt-br/default.htm


=
Instruções para entrar na lista, sair da lista e usar a lista em
http://www.mat.puc-rio.br/~nicolau/olimp/obm-l.html
=


[obm-l] Problema

2007-11-16 Thread Antonio Manuel Castro del Rio
Alguém teria a solução do seguinte problema.

Dois profissionais e cinco aprendizes, produzem 48 peças em 3 dias; um
profissional e um aprendiz produzem 45 peças em 9 dias. Quantas peças são
produzidas por 2 profissionais e 3 aprendizes em 5 dias? Resposta 60 peças.


[obm-l] Problema

2007-11-16 Thread Antonio Manuel Castro del Rio
 Alguém sabe resolver o problema, grato.

Considere-se um triângulo retângulo de hipotenusa a, sendo h a altura
relativa à hipotenusa e r o raio do círculo inscrito no triângulo,
inscrevem-se neste triângulo um quadrado de lados sobre os catetos e vértice
na hipotenusa e outro quadrado de lado sobre a hipotenusa e vértices sobre
os catetos. A razão entre as medidas dos lados do primeiro quadrado e do
segundo quadrado é:
Resposta:  a + r / a + 2r


[obm-l] Problema

2007-11-23 Thread Antonio Manuel Castro del Rio
Alo amigos, alguém pode me ajudar a resolver o seguinte problema:


Num teatro, quando o preço do ingresso para um espetáculo é P, o numero de
espectadores que a ele assiste é E. Para cada redução R no preço do
ingresso, há um aumento de espectadores Y. Para que a receita do espetáculo
seja máxima, o ingresso deve ter o seguinte preço:

Resposta: P/2 + ER/2Y


[obm-l] Problema!!

2009-03-23 Thread Antonio Manuel Castro del Rio
Ola, boa noite.
Preciso de ajuda para resolver um problema.

  COMO FAZER 96 VIRAR UMA SOMA DE DOIS QUADRADOS?

Desde já, obrigado
   Antonio del Rio


[obm-l] Problema

2002-12-09 Thread Cláudio \(Prática\)



Caros amantes da matematica:
 
O problema a seguir consta de uma lista de 
problemas preparatorios para a IMO:
 
 
Prove que existe uma bijecao  f: N --> N (N 
= conjunto dos numeros inteiros positivos) tal que:
 
Para todo inteiro positivo n, n divide f(1) + f(2) 
+ ... + f(n).
 
 
A minha ideia para este problema e definir 
indutivamente a funcao f da seguinte forma:
 
f(1) = 1
 
f(n+1) = menor inteiro positivo diferente de f(1), 
...,f(n) tal que n+1 divide f(1)+f(2)+...+f(n+1).
 
Assim, temos que: f(2)=3, f(3)=2, f(4)=6, f(5)=8, 
f(6)=4, 
 
Fui capaz de provar que f e bem definida. Por 
construcao, f e claramente injetiva e satisfaz a condicao de 
divisibilidade.
 
Entretanto, nao consigo demonstrar que f e 
sobrejetiva. Alguem tem alguma ideia?
 
 
Um abraco,
Claudio Buffara.


[obm-l] Problema

2002-12-17 Thread Tertuliano Carneiro de Souza Neto
Alguém pode me ajudar no seguinte problema?



O departamento de matemática tem 8 professores
graduados ocupando o mesmo gabinete. Cada um tanto
estuda em casa como no gabinete. Quantas escrivaninhas
deve haver no gabinete de modo que cada um tenha uma
pelo menos 90% do tempo?

Grato.
Tertuliano Carneiro,
De Salvador.

___
Yahoo! GeoCities
Tudo para criar o seu site: ferramentas fáceis de usar, espaço de sobra e acessórios.
http://br.geocities.yahoo.com/
=
Instruções para entrar na lista, sair da lista e usar a lista em
http://www.mat.puc-rio.br/~nicolau/olimp/obm-l.html
O administrador desta lista é <[EMAIL PROTECTED]>
=



[obm-l] Problema

2003-01-13 Thread ciceroth

OLa galera,

Estou enviando um bom problema de matriz. La vai...
Seja  A uma matriz nxn. Prove que se A^(n+1) = 0, então A^n = 0.

Cícero Thiago





--
Use o melhor sistema de busca da Internet
Radar UOL - http://www.radaruol.com.br



=
Instruções para entrar na lista, sair da lista e usar a lista em
http://www.mat.puc-rio.br/~nicolau/olimp/obm-l.html
O administrador desta lista é <[EMAIL PROTECTED]>
=



[obm-l] Problema

2003-01-17 Thread bene




  

  
Nos festejos juninos, 20 casais de dançarinos são 
colocados em círculo de tal maneira que um homem e uma mulher formando 
um par estão situados diametralmente opostos. Durante a dança, dois 
dançarinos adjacentes trocam de lugar enquanto todos os outros 
permanecem na mesma posição. Essa mudança é repetida com pares 
adjacentes até que, na posição final, os dois dançarinos de cada par 
estejam novamente diametralmente opostos, mas na posição contrária da 
inicial. Então o número mínimo de mudanças, de dois dançarinos 
adjacentes, para acontecer isso é:
(a) 20! (b) 100 (c) 10! (d) 19! (e) 
20


[obm-l] problema

2003-02-07 Thread elton francisco ferreira
Se o nó é igual a uma milha marítima por hora e uma
milha marítima é igual a 1852 m; quanto tempo uma
embarcação com velocidade constante de 10 nós gasta
para percorrer 370,4 Km?

___
Busca Yahoo!
O melhor lugar para encontrar tudo o que você procura na Internet
http://br.busca.yahoo.com/
=
Instruções para entrar na lista, sair da lista e usar a lista em
http://www.mat.puc-rio.br/~nicolau/olimp/obm-l.html
O administrador desta lista é <[EMAIL PROTECTED]>
=



[obm-l] problema

2003-02-24 Thread elton francisco ferreira
Dos 800 sargentos formados pela EsSa a cada ano, 5%
pedem para sair do exército ao completarem 5 anos de
serviço, a quantidade de sargentos formados pela EsSa
após 12 anos e que estão em atividade é?

___
Busca Yahoo!
O serviço de busca mais completo da Internet. O que você pensar o Yahoo! encontra.
http://br.busca.yahoo.com/
=
Instruções para entrar na lista, sair da lista e usar a lista em
http://www.mat.puc-rio.br/~nicolau/olimp/obm-l.html
O administrador desta lista é <[EMAIL PROTECTED]>
=


[obm-l] Problema

2003-03-23 Thread Claudio Buffara
Caros colegas da lista:

Aqui vai um que esta dando trabalho:

O conjunto {1,2,...,1978} eh particionado em 6 subconjuntos. Prove que um
destes subconjuntos contem um elemento que eh igual a soma de dois elementos
(nao necessariamente distintos) deste mesmo subconjunto.

Agradeco qualquer ajuda.

Um abraco,
Claudio.

=
Instruções para entrar na lista, sair da lista e usar a lista em
http://www.mat.puc-rio.br/~nicolau/olimp/obm-l.html
O administrador desta lista é <[EMAIL PROTECTED]>
=


[obm-l] problema

2003-06-06 Thread Ricardo Prins



é, morgado, não consegui. desisto. 
 
prove que, se b(1),b(2),b(3),...,b(n) é uma 
reordenação dos números positivos a(1),a(2),...,a(n), então
 
b(1)/a(1) + b(2)/a(2) + ... + b(n)/a(n) >= 
n
 
bom, a dica foi usar desigualdade das 
médias...tá... somatório dos a(i)/n >= raiz enésima do produtório dos 
a(i)...mas não consigo pensar em mais nadatentei indução tb não saiu...o que 
faço?


[obm-l] Problema

2003-05-29 Thread Fernando



 

Gostaria de ajuda para o seguinte 
problema:
 1) Mostrar que,se no sistema de axiomas 
substituirmos a propriedade cancelativa da Multiplicação( a(diferente de 0) , se 
ab=ac, então b=c) pela proposição (a.b=0.Então, a=0 ou b=0),  a propriedade 
cancelativa da multiplicação pode ser demonstrada a partir de novo sistema de 
axiomas. 
 
Atenciosamente, 
Fernando. 


[obm-l] Problema

2003-05-31 Thread Fernando



Gostaria de ajudar o para o seguinte problema:
Mostrar que:
/x/ >/ = 0 e /x/=0 se e somente se x= 0
Atenciosamente,
Fernando.


[obm-l] Problema

2003-06-02 Thread Fernando



 
 
Gostaria de ajudar o para o seguinte problema:
Mostrar que:
se o inteiro n>/ 3, então ( n!)^2 > n^n
Atenciosamente,
Fernando.


[obm-l] Problema

2003-07-31 Thread amurpe

Oi Pessoal, como posso resolver este problema ?

 Obrigado e um abraço.

Amurpe.

 
Em uma reunião há 201 pessoas de 5 nacionalidades 
diferentes. Sabe-se que, em cada grupo de 6, ao menos 2 
tem a mesma idade. 

Demonstrar que ha ao menos 5 pessoas do mesmo pais, da 
mesma idade e do mesmo sexo.


 
__
Acabe com aquelas janelinhas que pulam na sua tela.
AntiPop-up UOL - É grátis!
http://antipopup.uol.com.br/


=
Instruções para entrar na lista, sair da lista e usar a lista em
http://www.mat.puc-rio.br/~nicolau/olimp/obm-l.html
=


[obm-l] Problema

2003-08-04 Thread Benedito



Problema
Mostre que o abaixo é divisível por 
2003:
1*2*3*...*1001 + 1002*1003*...*2002.
 
Benedito Freire


[obm-l] Problema

2002-02-02 Thread Jose Paulo Carneiro
Title: Help



Uma professora me mencionou um jogo que eh jogado em alguns lugares do Rio 
(por exemplo, no Colegio Militar). Vou descrever:
O jogo eh jogado por 4 pessoas (apos a descricao, eh facil ver que tambem 
pode ser jogado por 3 ou 2 pessoas) em uma mesa retangular, onde faca sentido 
falar na "vertical", que varia com cada jogador. 
Um tabuleiro de 8 x 8 casas (naturalmente, pode-se pensar em n x n) eh 
colocado com as arestas paralelas as da mesa.
No tabuleiro, colocam-se ao acaso numeros inteiros em todas as casas, 
exceto em uma, onde se coloca uma estrela.
Comecando agora de um primeiro jogador e prosseguindo (digamos) em sentido 
horario, cada jogador retira (e guarda) um numero situado na mesma vertical (em 
relacao a este jogador) em que estah a estrela, e coloca a estrela no lugar 
do numero retirado.
O jogo acaba quando nao restam mais numeros no tabuleiro, e ganha aquele 
que tiver guardada a maior soma dos numeros retirados.
A consulta que faco, em nome desta professora, eh: este jogo tem 
estrategia vencedora?
JP


[obm-l] problema

2002-02-14 Thread Carlos Frederico Borges Palmeira

um problema aparentemente simples: considere uma elipse de semi-eixos a
,b, centrada em um ponto (c,d) do plano. Determinar condicoes em a,b,c,d
para que a elipse esteja toda contida no interior do disco x^2+y^2 <1.

Fred palmeira

=
Instruções para entrar na lista, sair da lista e usar a lista em
http://www.mat.puc-rio.br/~nicolau/olimp/obm-l.html
O administrador desta lista é <[EMAIL PROTECTED]>
=



[obm-l] Problema

2002-03-06 Thread André

Saudacoes, 

Alguem pode me ajudar na solucao do problema abaixo? 

Dado um triangulo ABC, seja AA' e CC' as bissetrizes dos angulos angA e angC 
respectivamente. Sabendo que angA > angB > angC e AA' = CC', prove que: 

c*sen((angA-angB)/2)=a*sen((angB-angC)/2). 

Agradeco... 

Andre/RJ. 

_
Oi! Você quer um iG-mail gratuito?
Então clique aqui: http://registro.ig.com.br/censo/igmail

=
Instruções para entrar na lista, sair da lista e usar a lista em
http://www.mat.puc-rio.br/~nicolau/olimp/obm-l.html
O administrador desta lista é <[EMAIL PROTECTED]>
=



[obm-l] Problema.

2002-03-09 Thread Moacyr Moreira

Bom dia,

 Se alguém puder me ajudar a resolver este
problema, eu ficaria muito grato.

 Problema: A Sra. Barnabé, proprietária de um sítio em
Arceburgo, notou que o capim crescia no pasto todo com
igual rapidez e espessura. Observando o consumo de
capim pela criação de cabras, a Sra. Barnabé notou que
50 cabras consumiam um pasto inteiro em 20 dias e que
40 cabras levaram 30 dias para consumir um pasto
inteiro. A partir desses dados, a Sra. Barnabé quer
determinar qual o número de cabras necessárias para
consumir um pasto inteiro em 40 dias. Resolva o
problema.

 Abraço,

Moacyr.

___
Yahoo! Empregos
O trabalho dos seus sonhos pode estar aqui. Cadastre-se hoje mesmo no Yahoo! Empregos 
e tenha acesso a milhares de vagas abertas!
http://br.empregos.yahoo.com/
=
Instruções para entrar na lista, sair da lista e usar a lista em
http://www.mat.puc-rio.br/~nicolau/olimp/obm-l.html
O administrador desta lista é <[EMAIL PROTECTED]>
=



[obm-l] problema

2002-03-17 Thread luizhenriquerick

Olá amigos da lista , se possível me ajudem com esse problema abaixo , desde
ja , grato.
Rick

Suprima cem díditos do número 1234567891011121314151617...5960 de modo a
obter o menor número possível . A seguir , refaça o mesmo para obter o maior
número possível . A soma dos algarismos desses dois números é:




--
Use o melhor sistema de busca da Internet
Radar UOL - http://www.radaruol.com.br



=
Instruções para entrar na lista, sair da lista e usar a lista em
http://www.mat.puc-rio.br/~nicolau/olimp/obm-l.html
O administrador desta lista é <[EMAIL PROTECTED]>
=



[obm-l] Problema

2002-05-21 Thread Eduardo Quintas

resolva a equação :

x^(sqrt x) = 1/2

PS.: x elevado a raiz quadrada de x = 1/2

=
Instruções para entrar na lista, sair da lista e usar a lista em
http://www.mat.puc-rio.br/~nicolau/olimp/obm-l.html
O administrador desta lista é <[EMAIL PROTECTED]>
=



[obm-l] Problema

2004-07-07 Thread Korshinoi
Imagine um  quadrado ABCD  de lado a. Imagine agora dois móveis partindo do mesmo ponto A. O móvel 1 faz os caminhos AB, BC, CD , DA , AB. O móvel 2 percorre os caminhos AC, CA, AC, ou seja, só se move pela diagonal.  Com base nisso responda:
a) Se esses móveis tiverem mesma velocidade e sairem ao mesmo tempo do vértice A , eles se encontrarão?. Justifique sua resposta.
b) Dê as possibilidades de encontro , sabendo que a diferença entre os pontos de partida é inferior a 1 minuto.
c) Qual a diferença entre os tempos de partida do primeiro e do segundo móvel, sendo que o encontro entre eles dá-se em 1 ano. considere que a distância AB é percorrida em 1 minuto.
   Quem puder analisar...agradeço,
    Korshinói


[obm-l] Problema

2004-12-22 Thread =?iso-8859-1?q?Andr=E9=20S=20Cardoso?=
Esse problema não eh difícil de resolver mas, gostaria de saber há algum método geral de resolução´.
 
Encontre x:
 
2^x+3^(1/x)=1
 
E agora em função de a, b e c:
 
a^x+b^(1/x)=c
 
[]s
André Scaranto Cardoso
 
		Yahoo! Acesso Grátis - Internet rápida e grátis. Instale o discador do Yahoo! agora.

[obm-l] Problema

2005-02-15 Thread benedito




Quinze moedas de mesmo 
diâmetro são dispostas formando um triângulo eqüilátero. As faces de cada uma 
das moedas são pintadas ou de branco ou de preto. Prove que, qualquer que seja a 
pintura, existem três moedas de mesma cor cujos centros são vértices de um 
triângulo eqüilátero.--
Esta mensagem foi verificada pelo sistema de anti-virus e 
 acredita-se estar livre de perigo.



[obm-l] PROBLEMA!

2005-04-01 Thread Rafael Alfinito Ferreira
EU TENTEI, TENTEI E ATÉ AGORA NÃO ENTENDI
AÍ VAI:
DADOS 5 NÚMEROS, AS SOMA 2 A 2 SÃO: 0, 2, 4, 4, 6, 8, 9, 11, 13 E 15 
RESPECTIVAMENTE.
DETERMINE OS NÚMEROS.

DESDE JÁ AGRADEÇO.
_
MSN Messenger: converse online com seus amigos .  
http://messenger.msn.com.br

=
Instruções para entrar na lista, sair da lista e usar a lista em
http://www.mat.puc-rio.br/~nicolau/olimp/obm-l.html
=


[obm-l] problema

2005-06-20 Thread Guilherme Neves

Dado um triangulo qualquer ABC, e M , N e O os pés das alturas sobre os lados AB, AC e BC, respectivamente. Se o angulo ABN vale 40º, quanto vale o angulo MON?MSN Busca: fácil, rápido, direto ao ponto.  Encontre o que você quiser. Clique aqui. 

=
Instruções para entrar na lista, sair da lista e usar a lista em
http://www.mat.puc-rio.br/~nicolau/olimp/obm-l.html
=


[obm-l] Problema

2005-06-25 Thread Lincoln



Alguém pode me dar uma ajuda neste 
problema?
 

Seja 
ABCD um retângulo de lados AB = 4 e BC =3.  A perpendicular à diagonal BD traçada por A corta BD no ponto H. Chamamos de M o ponto médio de BH e de N o ponto médio de CD. Calcule a medida do segmento MN. 
 
 


[obm-l] Problema

2009-07-11 Thread Antonio Manuel Castro del Rio
Alguém poderia me ajudar na resolução de um problema.
" Com as 10 primeiras letras do alfabeto, quantos anagramas posso formar, de
modo que apareçam pelo menos 2 dessas letras: A, B e C.?"
Desde já agradeço a ajuda.
 Antonio del Rio


[obm-l] Problema

2009-09-22 Thread Luís Eduardo Háteras

Olá pessoal,

 

Sou novo nessa lista e estou com dúvida nesse exercício, alguém saberia 
resolver ? E alguém sabe como me explicar porque não consegui compreender como 
resolver.

 

Obrigado.

 

PROBLEMA:

 

O Daniel e o Bruno estao numa estaçao `a espera de um comboio. Para se 
entreterem,
decidem calcular o comprimento de um comboio de mercadorias que passa
pela estação sem alterar a velocidade. Quando a frente do comboio passa por eles
(o Daniel e o Bruno estao no mesmo lugar), o Daniel começa a andar no sentido do
movimento do comboio e o Bruno começa a andar no sentido oposto. Os dois 
caminham
`a mesma velocidade e cada um deles para no momento em que se cruza com
o fim do comboio. O Daniel andou 45 metros e o Bruno 30. Qual é o comprimento
do comboio?

(A) 50 metros (B) 75 metros (C) 120 metros (D) 180 metros (E) 210
metros


  
_
Novo Internet Explorer 8. Baixe agora, é grátis!
http://brasil.microsoft.com.br/IE8/mergulhe/?utm_source=MSN%3BHotmail&utm_medium=Tagline&utm_campaign=IE8

[obm-l] Problema

2009-09-24 Thread Paulo Barclay Ribeiro
Prezados, 
 
Peço uma ajuda (orientação)na resolução do seguinte problema:
Qual o valor da soma de todos os numeros naturais de três algarismos?
 
Desde  já agradeço a gentileza
 
Paulo Barclay 


  

Veja quais são os assuntos do momento no Yahoo! +Buscados
http://br.maisbuscados.yahoo.com

[obm-l] Problema

2009-10-29 Thread Benedito
Problema
Seja  n um número inteiro positivo.
Encontre o número máximo de triângulos não congruentes cujos lados tem 
comprimentos inteiros menores do que ou iguais a n.



[obm-l] problema

2010-10-05 Thread Vinícius
parece fácil, mas eu empaquei...

Cada cartela de uma coleção é formada por seis quadrados coloridos,
justapostos
como indica a figura abaixo.
Em cada cartela, dois quadrados foram coloridos de azul,
dois de verde e dois de rosa. A coleção apresenta todas as
possibilidades de distribuição dessas cores nas cartelas nas
condições citadas e não existem cartelas com a mesma
distribuição de cores. Retirando-se ao acaso uma cartela da
coleção, a probabilidade de que somente uma coluna
apresente os quadrados de mesma cor é de:


[obm-l] Problema

2012-02-05 Thread Henrique Rennó
Oi, boa tarde.

A solução do problema 1 da primeira fase do nível universitário na eureka
34 página 60 (
http://www.obm.org.br/export/sites/default/revista_eureka/docs/eureka34.pdf)
apresenta a função de posição considerando tempos inicial -1 e final 0,
sendo a função do objeto mais rápido dada por B(t) = (1,1,1) +
rq(3)*t*(1,1,1), onde rq é a raíz quadrada. Considerando t = -1 na equação,
temos B(-1) = (1-rq(3), 1-rq(3), 1-rq(3)), que é diferente da posição
inicial (0,0,0). Outra dúvida é como ficariam as funções se considerarmos
como tempo inicial e final os valores 0 e 1, respectivamente.

Obrigado

-- 
Henrique


[obm-l] Problema

2012-02-07 Thread Bob Roy
0lá ,

Poderiam me ajudar na questão  a seguir  ?

Em uma circunferência colocamos os números 5, 2,3,0, 5 e 6 (por exemplo
nesta ordem  no sentido horário) .A cada momento escolho  um número
qualquer e adiciono uma unidade a ele e aos dois vizinhos .É posível em
algum momento todos ficarem iguais ? se possível , como devemos proceder ?

Agradeço desde já qualquer  resposta

Bob


[obm-l] Problema

2012-02-19 Thread Henrique Rennó
Oi, bom dia.

No problema 2 do nível universitário da XXXII olimpíada brasileira de
matemática que está na Eureka! 34, por que se existem Np pares ordenados
(x, y), onde x, y pertence a {0, 1, 2, ..., p-1} e o número K = 5x^2 + 7y^2
- 1 é divisível por p, se p for 143 = 11.13, então o número de pares
ordenados é N11.N13? N11 não são os pares em que K é divisível apenas por
11 e N13 apenas por 13? Multiplicando os dois, temos os números de pares em
que K é divisível por 143? Por quê?

Obrigado

-- 
Henrique


[obm-l] Problema

2012-06-02 Thread benedito
Problema

Temos N varas azuis e N varas vermelhas. A soma dos comprimentos de todas as 
varas azuis é igual à soma dos comprimentos de todas as varas vermelhas. 
Sabe-se que é possível construir um polígono de N lados usando todas as varas 
azuis e também é possível construir um polígono de N lados com todas as varas 
vermelhas.

Determinar se é sempre possível escolher uma vara azul e uma vermelha, mudar 
suas respectivas cores, de modo que seja novamente possível construir um 
polígono de N lados com as varas azuis e um polígono de N lados com as varas 
vermelhas.

Resolver o problema para

(a) N = 3.   (b) N arbitrário maior do que 3


[obm-l] PROBLEMA

2012-12-05 Thread Luís Lopes

Sauda,c~oes, 

Mais uma tentativa de ver esta mensagem chegar na lista. 

O Bernardo já respondeu numa troca de emails particular. 

Esta mensagem deve ser lida de baixo para cima. 

Abraços, 
Luís 



O arquivo está em 
http://www.escolademestres.com/download/questao_luis_lopes_20121205.pdf 

> Date: Wed, 5 Dec 2012 09:59:41 -0500
> Subject: Re: [obm-l] RE: PROBLEMA
> From: bernardo...@gmail.com
> To: qed_te...@hotmail.com
> 
> 2012/12/5 Luís Lopes :
> > Oi Bernardo,
> >
> > Não consigo me comunicar com a lista. Mas pelo jeito a
> > lista (você) recebe(u) minhas msgs.
> Estranho.
> 
> > Tenho recebido somente algumas msgs da lista. Num outro
> > email (outra conta) acho que recebo todos. Pode ser problema
> > com o hotmail também, sei lá.
> Pode ser...
> 
> > Tudo começou com o email abaixo, que mandei pra lista:
> Pode ser uma questão de PDFs. Sei lá. Pode ser mesmo "esse PDF em particular".
> 
> Para responder a pergunta: todas as afirmações são verdadeiras, é
> claro. O problema da "d" é que a projeção estereográfica envia
> círculos passando pelo polo Norte em retas do plano + ponto no
> infinito. Mas como f está definida apenas na esfera - polo Norte,
> nenhuma dessas circunferências serve, porque f não aplica o polo norte
> em lugar nenhum. Logo, as circunferências que você *pode* usar são
> realmente mandadas em círculos do plano.
> 
> Ah, óbvio que isso é um problema de geometria complexa, mas fica muito
> mais legal retirar todo o contexto e fazer uma questão absurda de
> vestibular. Pena, né.
> -- 
> Bernardo Freitas Paulo da Costa

From: qed_te...@hotmail.com
To: obm-l@mat.puc-rio.br
Subject: FW: PROBLEMA
Date: Mon, 3 Dec 2012 12:44:01 +





Sauda,c~oes, 

Mais uma vez, peço a ajuda de vocês para uma resposta 
mais completa e interessante. 

Espero que o anexo passe. Se não, alguém poderia colocá-lo 
disponível em algum site? 


Subject: PROBLEMA
Date: Mon, 3 Dec 2012 08:17:24 -0300








Prezado Luís, 



Gostaria de sua ajuda para o problema(conforme 
arquivo PDF anexo). 
Se possível, gostaria que justificasse porque os demais 
itens são verdadeiros. 


Abraços, 

FERNANDO FORTALEZA-CE. 

PS: E eu gostaria de saber também por que o item falso é falso. 
Obrigado. 

Abraços, 
Luís 

  

[obm-l] problema

2013-02-15 Thread grego
Olá, companheiros!
Um aluno me perguntou o seguinte:
a <=b<=c<=d
1/a+1/b+1/c+1/d=1
Quantas quádruplas ordenadas (a, b, c, d) de naturais satisfazem a igualdade?
Um abraço!
Grego

[obm-l] Problema

2013-03-22 Thread Benedito
Problema

Dois pontos, M e Q, são escolhidos aleatoriamente num disco unitário, mas em
regiões opostas, determinadas por um diâmetro AB. 

Qual é a probabilidade de que a distância entre M e Q seja menor do que 1?



[obm-l] problema

2013-12-13 Thread saulo nilson
encontre todas as soluçoes de 2^x=x

-- 
Esta mensagem foi verificada pelo sistema de antivírus e
 acredita-se estar livre de perigo.



[obm-l] Problema

2005-09-16 Thread Rejane



Bom dia,
 
Alguém pode me ajudar com esse 
probleminha?
 
Um datilografo A pode fazer um trabalho em 12 horas 
e o datilógrafo B, em 18 horas.  Os dois juntos, poderão realizar o mesmo 
trabalho no seguinte tempo:
 
a) 7h 
12minb) 7h 
30min    c) 8h 
24min    d) 8h 
30min    e) 9h 16min
 
Obrigada,
 
 


[obm-l] problema

2005-10-30 Thread elton francisco ferreira
Ao analisar notas fiscais de uma firma, o auditor
deparou-se com a seguinte situação:


QuantidadeMercadoria   Preço unitário   Total (R$)
*metros  cetim 21,00  *56,00

Não era possível ver o numero de metros vendidos, mas
sabia-se que era um numero inteiro. No valor total, so
xapareciam os dois ultimos dos três algarismos da
parte inteira. Com as afirmações acima, o auditor
concluiu que a quantidade de cetim, em metros,
declarada nessa nota foi:

16
26
36
46
48








___ 
Promoção Yahoo! Acesso Grátis: a cada hora navegada você
acumula cupons e concorre a mais de 500 prêmios! Participe!
http://yahoo.fbiz.com.br/
=
Instruções para entrar na lista, sair da lista e usar a lista em
http://www.mat.puc-rio.br/~nicolau/olimp/obm-l.html
=


[obm-l] Problema

2005-12-28 Thread garcia

Me lembrei de outro velho problema que me passaram com dados novos:

Um gênio matemático recebe, num papel, a soma de dois números inteiros entre 2 e
100. Um outro gênio recebe o produto dos mesmos dois números. Os dois iniciam o
diálogo:
   
- Este produto não é o suficiente para achar os dois números.
- Eu sabia.
- Então, eu conheço estes números.
- Nesse caso, eu também.
- Quais são os dois números?





Citando Adriano Torres <[EMAIL PROTECTED]>:

> Olá, sou novo aqui na lista, e gostaria de propor um problema para que me 
> ensinassem a solução.
> Estou enviando a figura do triângulo para que possa ser visto.
> É um triangulo isóceles, com AB = AC, ângulo bÂc = 20°, cBt = 30° e bTc 
> reto. Determinar o angulo cPq. Ficarei grato se souber a solução, há muito 
> tento e nao consigo resolver.
> Desculpa pela má qualidade da imagem, a fiz no paint, nao tenho muita 
> habilidade.
> Obrigado,
>  Adriano Torres
> 
> 
> =
> Instruções para entrar na lista, sair da lista e usar a lista em
> http://www.mat.puc-rio.br/~nicolau/olimp/obm-l.html
> =
> 





This message was sent using IMP, the Internet Messaging Program.
=
Instruções para entrar na lista, sair da lista e usar a lista em
http://www.mat.puc-rio.br/~nicolau/olimp/obm-l.html
=


[obm-l] problema

2006-01-17 Thread elton francisco ferreira
Determine a e b, de modo que -3 e 2 sejam raízes da
equação ax^2 - bx + (a + b + 2) = 0.








___ 
Yahoo! doce lar. Faça do Yahoo! sua homepage. 
http://br.yahoo.com/homepageset.html 

=
Instruções para entrar na lista, sair da lista e usar a lista em
http://www.mat.puc-rio.br/~nicolau/olimp/obm-l.html
=


[obm-l] Problema

2006-05-25 Thread Bené

Problema
Um matemático sai de casa todos os dias com duas caixas de fósforos, cada 
uma contendo  45  palitos. Toda vez que ele quer acender um cigarro, pega, 
ao acaso, uma das caixas e retira de lá um palito. O matemático é muito 
distraído, de modo que quando ele retira o último palito de uma caixa, não 
percebe que a caixa fica vazia. Como ele fuma fuma muito, em certa hora ele 
pega uma caixa e constata que ele está vazia.
Qual é a probabilidade de nesse momento a outra caixa conter exatamente  10 
palitos?


Benedito 


=
Instruções para entrar na lista, sair da lista e usar a lista em
http://www.mat.puc-rio.br/~nicolau/olimp/obm-l.html
=


[obm-l] Problema

2006-05-28 Thread benedito




Problema
Um matemático sai de casa todos os dias com duas caixas de fósforos, cada 
uma contendo  45  palitos. Toda vez que ele quer acender um cigarro, pega 
uma das caixas, ao acaso,e retira de lá um palito. O matemático é muito 
distraído, de modo que quando ele retira o último palito de uma caixa, não 
percebe que a caixa fica vazia. Como ele fuma muito, em certa hora ele pega 
uma caixa e constata que ela está vazia.
Qual é a probabilidade de nesse momento a outra caixa conter exatamente  10 
palitos?


Benedito


=
Instruções para entrar na lista, sair da lista e usar a lista em
http://www.mat.puc-rio.br/~nicolau/olimp/obm-l.html
=


[obm-l] problema

2006-06-23 Thread elton francisco ferreira
Eduardo tem 1.325,00 reais e Alberto 932,00. Aquele
economiza 32,90 reais por mês e este, 111,50 por mês.
Depois de quanto tempo terão quantias iguais?



___ 
Abra sua conta no Yahoo! Mail: 1GB de espaço, alertas de e-mail no celular e 
anti-spam realmente eficaz. 
http://mail.yahoo.com.br/
=
Instruções para entrar na lista, sair da lista e usar a lista em
http://www.mat.puc-rio.br/~nicolau/olimp/obm-l.html
=


[obm-l] Problema

2006-07-15 Thread Natan Padoin
Olá pessoal da lista, alguém pode me mostrar a solução do seguinte problema:     A partir da função dada, h(d)=  –2d^2 –8d –1, onde d é a largura e h a altura de um túnel a ser construído, ambos são dados em metros,
 qual a altura máxima da cobertura parabólica que esse túnel pode ter?     A resposta correta é 7m. Se possível, mandem a resposta para o e-mail [EMAIL PROTECTED]    Desde já agradeço a atenção. Abraço. 
		 
Abra sua conta no Yahoo! Mail - 1GB de espaço, alertas de e-mail no celular e anti-spam realmente eficaz. 

[obm-l] problema

2006-08-13 Thread elton francisco ferreira
uma impressora tem capacidade para imprimir 14 páginas
por minuto em preto e 10 páginas por minuto em cores.
Quanto tempo outra impressora levaria para imprimir um
texto com 210 páginas em preto e 26 em cores se sua
capacidade de operção é igual a 80% da capacidade da
primeira?





___ 
Novidade no Yahoo! Mail: receba alertas de novas mensagens no seu celular. 
Registre seu aparelho agora! 
http://br.mobile.yahoo.com/mailalertas/ 
 

=
Instruções para entrar na lista, sair da lista e usar a lista em
http://www.mat.puc-rio.br/~nicolau/olimp/obm-l.html
=


[obm-l] Problema

2006-10-10 Thread Bruno Carvalho
Amigos peço ajuda para os seguintes problemas:     1)Demonstrar que os números 49 , 4489, 444889 obtidos colocando o número 48 no meio do número anterior,são quadrados de números inteiros.     2) n retas paralelas de um plano se cruzam por uma série de m retas paralelas. Quantos paralelogramos podem ser separados na rede obtida ?     mais uma vez, obrigado.     Bruno 
		 
O Yahoo! está de cara nova. Venha conferir!

[obm-l] Problema

2007-01-29 Thread Carlos Gomes
Vê se alguém tem alguma sugestão para essa questao;

Disponha em linha reta, numa ordem, os números inteiros de 1 até 49, de modo 
que o valor absoluto da diferença de quaisquer dois vizinhos, nessa ordem, seja 
ou 7 ou 9.



Obg

C.Gomes


[obm-l] problema

2003-10-11 Thread Marcelo Souza
Alguém poderia me ajudar

O numero natural n tem seus divisores x1,x2,x3...,xk ordenados de forma que 
x1

[]'s

_
MSN Messenger: instale grátis e converse com seus amigos. 
http://messenger.msn.com.br

=
Instruções para entrar na lista, sair da lista e usar a lista em
http://www.mat.puc-rio.br/~nicolau/olimp/obm-l.html
=


[obm-l] Problema

2003-10-12 Thread Daniel Melo Wanzeller



Pessoal,
 
    Gostaria de saber se alguem tem 
a solucao da seguinte equacao:
 
x^x^x = 2^ [-(sqrt 2)]. 
 
Peguei esse problema na internet e a solucao 
apresentada nao confere com a minha. Lá a solucao é 1/2. Se alguem poder me 
ajudar agradeço.
 
[]´s Daniel


[obm-l] Problema

2003-10-24 Thread Tiago Carvalho de Matos Marques
Ola!!

Podem, por favor, me ajudar?
E algo meio ridiculo pra voces, mas...peco que tenham paciencia comigo. =]

Nao consegui descobrir uma logica de resolucao.
Nao lembro exatamente os valores, mas nao se importem muito com os valores, oq quero 
entender e a logica.
Se nao fizer muito sentido com esses valores (ou se ficar muito facil), por favor, se 
possivel facam as devidas alteracoes, uma solucao generica...

Um caminhao pode levar ate 1500kg.
Temos disponiveis 80 caixas de uva e 80 de maca.
Caixa de uva vale 1real, de maca 0,25.
Caixa de uva pesa 20kg de maca pesa 15.
Quantas caixas de cada levar para receber o maximo possivel?

Muito obrigado!
[]s!
=
Instruções para entrar na lista, sair da lista e usar a lista em
http://www.mat.puc-rio.br/~nicolau/olimp/obm-l.html
=


[obm-l] Problema

2003-11-24 Thread Benedito






Um problema da Olimpíada Espanhola, se não me engano, de 1985:
 
Para cada número natural n, o número (n+1)(n+2)(n+3)...(2n)  é divisível por (2 elevado a n).
Benedito







  IncrediMail - O mundo do correio eletrônico finalmente desenvolveu-se - Clique aqui

[obm-l] Problema

2003-11-26 Thread Benedito






Problema
Use um argumento combinatório para mostrar que o número (n^2)!  é divisível por
(n!)^(n+1).
 
Benedito
 







  IncrediMail - O mundo do correio eletrônico finalmente desenvolveu-se - Clique aqui

[obm-l] Problema

2004-02-10 Thread benedito
De uma prova da olimpíada chinesa (1986/1987), um problema interessante:

A soma de  m  inteiros positivos pares e  n  inteiros positivos ímpares  é
igual  1987.
Qual é o valor máximo de  3m + 4n?

Benedito

=
Instruções para entrar na lista, sair da lista e usar a lista em
http://www.mat.puc-rio.br/~nicolau/olimp/obm-l.html
=


[obm-l] Problema

2004-02-15 Thread benedito

> Prezado Cláudio,
>
> Do mesmo jeito que você começou, vi uma solução assim:
>
> 1987 = (2 + 4 + ..+ 2m) + (1 + 3 + 5 + ... + 2m - 1) = 1987, ou ainda
> (m + 1/2)^2  +  n^2 = 1987 + 1/4.
>
> Usando a desigualdade de Cauchy:
> 3m + 4n) = 3(m + 1/2) + 4n -3/2 <= (3^2 ^4^2)^1/2 [(m +1/2)^2 +
^2]^1/2  -
> 3/2
> <= 5 (1987 + 1/4)^1/2 - 3/2.
> Portanto, 3m + 4n < 222. Logo,  3m + 4n <= 221.
> Os   27  números  pares   2, 4, 6, ..., 50, 52, 60  e  35  ímpares  1, 3,
> ..., 69  realizam o máximo
> 3m + 4n = 221.
>
Oi, Benedito:

Realmente essa desigualdade nao tinha nem passado pela minha cabeca.

O mais interessante eh o fato da solucao otima nao consistir de numeros
pares consecutivos. Acho que problemas de otimizacao com inteiros as vezes
tem este tipo de surpresa - esse eh o moral da historia.

Outra solucao otima (desta vez com impares nao consecutivos) seria:
Pares (27): 2, 4, 6, , 50, 52, 54
Impares (35): 1, 3, 5, ..., 65, 67, 75.

Mande a solucao pra lista, ou pelo menos pro Artur.

Um abraco,
Claudio.


--
Esta mensagem foi verificada pelo sistema de antivírus e
 acredita-se estar livre de perigo.



=
Instruções para entrar na lista, sair da lista e usar a lista em
http://www.mat.puc-rio.br/~nicolau/olimp/obm-l.html
=


[obm-l] Problema

2004-03-18 Thread benedito



Problema
Um jogo entre duas pessoas, A  e  B, é definido da seguinte 
maneira:
A escolhe um intervalo fechado arbitrário J1, de comprimento menor do 
que 1;  
A seguir, B escolhe um intervalo fechado qualquer J2  contido 
em  J1, de comprimento menor do que 1/2.
Na sua vez de jogar, A escolhe um intervalo fechado qualquer contido 
em  J2, de comprimento  1/3; e assim por diante.
Pelo princípio dos intervalos encaixantes, a intereseção de  J1, J2, 
J3, ...  contém um único número real  r.
Se  r  for um número racional  A vence o jogo.
Se  r  for irracional, B vence o jogo.
Mostre que  A  tem uma estratégia vencedora, não importa 
com  B  jogue.
 
NOTA:1) Este problema aparece, pag. 157, problema 11, no excelente 
livro:
 Excursion  Calculus - An interplay of the continous and the 
discrete - Robert M. Young
The Mathematical Association of American.  1992.
2) Há tempos atrás, o Gugu apresentou-me uma solução muito interessante 
desse problema.
 
Benedito
 
<>

[obm-l]Problema

2004-05-27 Thread João Luís



Olá,
Desejo submeter um problema:
"Qual é a maior potência de 3 divisível pelo 
produto dos primeiros 300 naturais diferentes de zero?"
Qual seria o mais prático método de achar 
quantos fatores "3" estão contidos nesse produto?


[obm-l] Problema

2004-05-28 Thread Lista OBM
Gostaria de saber se alguém poderia me ajudar com o seguinte problema: 
Sejam A e B anéis ordenados. Diz-se que um homomorfismo injetivo f: A --> B preserva ordem se, para todo a > 0 em A, tivermos f(a) >; 0. Sejam K um corpo ordenado e f: Q --> K um homomorfismo injetivo dos números racionais em K. Mostre que, necessariamente, f preserva a ordem.
 
Grato desde já com a possível ajuda de vocês.Yahoo! Messenger - Fale com seus amigos online. Instale agora!

[obm-l] Problema

2014-10-16 Thread Mauricio Barbosa
Boa tarde amigos,
alguém poderia me ajudar com o problema:
Em um quadrilátero convexo de área 32cm2, a soma dos comprimentos de dois
lados opostos mais uma diagonal é 16 cm. Determine os valores possíveis
para a outra diagonal.
Obrigado!!!

-- 
Esta mensagem foi verificada pelo sistema de antiv�rus e
 acredita-se estar livre de perigo.



[obm-l] Problema

2015-06-22 Thread Douglas Oliveira de Lima
Olá caros colegas, gostaria de uma ajuda no seguinte problema:

Em uma reta há 1999 bolinhas. Algumas são verdes e as demais azuis(poderiam
ser todas verdes ou todas azuis). Debaixo de cada bolinha escrevemos o
número igual a soma da quantidade de bolinhas verdes à sua direita dela
mais a quantidade de bolinhas azuis a esquerda dela. Se, na sequência de
números assim obtida, houver exatamente três números que aparecem uma
quantidade ímpar de vezes, quais podem ser estes números?


Abraço


Douglas Oliveira

-- 
Esta mensagem foi verificada pelo sistema de antiv�rus e
 acredita-se estar livre de perigo.



[obm-l] Problema

2015-07-01 Thread benedito
Problema 
Dois jogadores, A e B, disputam um jogo, em que jogam alternadamente. O jogador 
A começa. Uma jogada consiste em apagar um dos números inteiros do conjunto {1, 
2, 3,..., 27} até que reste somente dois números. Se a soma desses dois últimos 
números for divisível por 5, o jogador A vence, caso contrário, vence o jogador 
B. 
Se cada jogador faz suas melhores jogadas, quem vence: A ou B? Qual é a 
estratégia para vencer? 

-- 
Esta mensagem foi verificada pelo sistema de antiv�rus e
 acredita-se estar livre de perigo.



[obm-l] Problema

2015-07-25 Thread Israel Meireles Chrisostomo
Seja um triângulo inscrito numa circunferência de raio r, e seja os lados
deste triângulo a,b,c.Seja uma esfera de raio 2r centrada nos pontos
(x_0,y_0,z_0) .Seja um ponto qualquer no espaço tridimensional dado pelas
coordenadas (x_i,y_ j,z_k) .Prove que dentre todas os valores das
coordenadas  (x_i,y_ j,z_k) que satisfazem |x_i-x_0|=a ,|y_ j-y_ 0|=b e
|z_k-z_0|  =c, existe apenas uma tripla de reais (a menos da ordem de
x_i,y_ j,z_k) que são coordenadas da superfície dessa esfera.

Alguém sabe alguma aplicação prática para este problema, isto é, alguém
pode me dar uma ideia interessante para contextualizar este problema?Além
disso, alguém pode confirmar para mim se este problema está formulado
corretamente?Se caso afirmativo, podem sugerir soluções?

-- 
Esta mensagem foi verificada pelo sistema de antiv�rus e
 acredita-se estar livre de perigo.



[obm-l] Problema

2016-07-08 Thread Lucas Melo
Ola, alguem poderia me ajudar nesse problema ?
E dada uma equacao do segundo grau x^2 + ax + b= 0 com raizes inteiras a1 e a2. 
Consideramos a equacao do segundo grau x^2 + a1x + b1=0. Se a equacao x^2 + a1x 
+ b1=0 tem raizes inteiras a2 e b2, consideramos a equacao x^2 + a2x + b2 = 0. 
Se a equacao x^2 + a2x + b2=0 tem raízes inteiras a3 e b3, consideramos a 
equacao  x^2 + a3x + b3=0 . E assim por diante ; se encontrarmos uma equacao 
sem raizes inteiras , encerramos o processo. Encontre todas as equacoes das 
quais podemos repetir o processo indefinidamente.



-- 
Esta mensagem foi verificada pelo sistema de antiv�rus e
 acredita-se estar livre de perigo.


=
Instru珲es para entrar na lista, sair da lista e usar a lista em
http://www.mat.puc-rio.br/~obmlistas/obm-l.html
=


[obm-l] Problema

2016-10-06 Thread regis barros
Boa noite pessoalSe y^x=2 e x^y=3, encontrar os valores de x e y?
Regis
-- 
Esta mensagem foi verificada pelo sistema de antiv�rus e
 acredita-se estar livre de perigo.



[obm-l] Problema

2016-10-08 Thread regis barros
Bom diasegue o problemase x^y = 2 e y^x = 3, encontrar os valores de x e y.
Grato
Regis 

Em Quarta-feira, 5 de Outubro de 2016 18:01, vinicius raimundo 
 escreveu:
 

 Obrigado Douglas 

Em quarta-feira, 5 de outubro de 2016, Douglas Oliveira de Lima 
 escreveu:

Bom vamos lá, não tem nada de bonito nessa resolução.
Seja O o centro do ex-incirculo de ABC tangente ao lado BC, temos que AO é 
bissetriz do ângulo BAC, seja Q a intercessão de AO com BC, e J o pé da 
perpendicular tirada de O ao lado AC, sendo BAQ=x, nós teremos CAQ=ACB=x, 
AQB=OQC=2x. E OC é bissetriz de BCJ, assim BCO=90-x/2, e sendo P a intercessao 
de MO com BC.
1)Aplicando lei dos senos no triângulo AQC teremos 
AQ/AC=senx/sen(2x)
2)Agora aplicando no triângulo AMO teremos 
AM/MO=sen(QOP)/senx
3)E no triângulo CMO novamente lei dos senos teremos
MC/MO=sen(COP)/cos(x/2)
4)Como AM=MC, dos itens (2) e (3) segue que 
sen(QOP)/sen(COP)=senx/cox(x/ 2)
5) Para o triângulo QPO, nós teremos 
sen(QOP)=[(QP)sen(2x)]/PO
6) Para o triângulo CPO, nós teremos 
sen(COP)=[(CP).cos(x/2)]/PO
7)Dos itens (5) e (6) podemos concluir que 
sen(QOP)/sen(COP)=[(QP).sen( 2x)]/[(CP).cos(x/2)]
8)E de (4) e (7) nós temos
senx/cos(x/2)=[(QP).sen(2x)]/[ (CP).cos(x/2)], ou melhor QP/CP=senx/sen(2x)
9)Agora de (1) e (8) AQ/AC=QP/CP, donde vem
QAP=CAP e BAP=x+QAP=x+CAP=BPA, ou seja ABP é isosceles e AB=BP.

Um abraço  do Douglas Oliveira.
Em 1 de outubro de 2016 19:54, vinicius raimundo  
escreveu:

Será que alguém poria me ajudar na seguinte questão?
   
   -  (Belarus) Seja O o centro do círculo ex-inscritodo triângulo ABC oposto 
ao vértice A. Seja M oponto médio de AC e seja P a intersec ̧ão das retasMO e 
BC. Prove que se ∠BAC = 2∠ACB, então  AB = BP.  

--
Esta mensagem foi verificada pelo sistema de antivírus e 
 acredita-se estar livre de perigo.


--
Esta mensagem foi verificada pelo sistema de antivírus e 
 acredita-se estar livre de perigo.

--
Esta mensagem foi verificada pelo sistema de antiv�us e 
 acredita-se estar livre de perigo.

   
-- 
Esta mensagem foi verificada pelo sistema de antiv�rus e
 acredita-se estar livre de perigo.



[obm-l] Problema

2017-04-15 Thread Luiz Antonio Rodrigues
Olá, pessoal! Bom dia! Eu resolvi o problema abaixo supondo que as vacas
comem uma vez por dia. Escrevi para perguntar se alguém consegue resolver
de outra forma. A resposta é 28 dias. Muito obrigado, um abraço e uma ótima
Páscoa para todos.
Um fazendeiro possui ração suficiente para alimentar suas 16 vacas durante
62 dias. Após 14 dias, ele vende 4 vacas. Passados mais 15 dias, ele compra
9 vacas. Quantos dias vai durar o restante da ração?

-- 
Esta mensagem foi verificada pelo sistema de antiv�rus e
 acredita-se estar livre de perigo.



[obm-l] Problema

2018-11-26 Thread benedito
Alguém pode me dar uma sugestão para o problema seguinte? 

Problema 
Há uma lâmpada em cada casa de um tabuleiro 2019 x 2019 . Cada lâmpada está 
acesa ou apagada. Uma lâmpada é chamada de ruim se ela tem um número par de 
vizinhas que estão acesas. 
Qual é o menor número possível de lâmpadas ruins no tabuleiro? 
(Duas lâmpadas são vizinhas se elas se encontram em casas do tabuleiro que 
compartilham um lado.) 

NOTA -Tentei raciocinar com o tabuleiro no qual as casas estejam pintadas 
alternadamente de branco e preto. Desse modo, pode-se ver que como as vizinhas 
de uma casa branca são todas pretas, parece que uma casa ruim branca não 
influencia outra branca. Parece que o mesmo deve acontecer com as casas pretas. 
No caso particular do tabuleiro 3x3, encontrei que o número procurado é 1: 
B P B 
P B P 
B P B 

Obrigado. 

Benedito Freire 


-- 
Esta mensagem foi verificada pelo sistema de antiv�rus e
 acredita-se estar livre de perigo.



[obm-l] PROBLEMA

2019-09-03 Thread benedito
Problema 


Um mágico e seu assistente realizam uma mágica da maneira seguinte. Há 12 
caixas vazias e fechadas, colocadas em fila. O mágico sai da sala e uma pessoa 
do público escolhe duas caixas e esconde em cada uma delas uma moeda, deixando 
a fila de caixas da mesma forma como era, mas o assistente sabe quais são as 
duas caixas que têm moedas. O mágico retorna para a sala e o assistente escolhe 
uma caixa que ele sabe que está vazia. Das restantes, o mágico então escolhe 
quatro caixas que são abertas simultaneamente. O objetivo do mágico é que, 
entre essas quatro caixas, duas contenham as moedas. 

Desenvolva um método que permita que o mágico e seu assistente realizem a 
mágica com sucesso. 

-- 
Esta mensagem foi verificada pelo sistema de antiv�rus e
 acredita-se estar livre de perigo.



[obm-l] Problema

2020-03-17 Thread benedito
Problema 
Um mágico e seu assistente realizam um truque da maneira seguinte. Existem 12 
caixas vazias e fechadas, colocadas em fila. O mágico sai da sala e uma pessoa 
do público escolhe duas caixas e esconde em cada uma delas uma moeda, deixando 
a fila de caixas da mesma forma como era, mas o assistente sabe quais são as 
duas caixas que têm as moedas. O mágico retorna para a sala e o assistente 
escolhe uma caixa que ele sabe que está vazia. Das restantes, o mágico então 
escolhe quatro caixas que são abertas simultaneamente. O objetivo do mágico é 
que, entre essas quatro caixas, duas contenham as moedas. 
Desenvolva um método que permita que o mágico e seu assistente realizem a 
mágica com sucesso 


-- 
Esta mensagem foi verificada pelo sistema de antiv�rus e
 acredita-se estar livre de perigo.



Re: [obm-l] Problema!

2007-09-03 Thread Rogerio Ponce
Ola' pessoal,

inicialmente, durante M horas (correspondentes a meio dia) , N pessoas 
trabalharam no campo grande. Em seguida, durante M horas, N/2 pessoas ainda 
trabalharam no campo grande. Enquanto isso, no campo pequeno, N/2 pessoas 
trabalharam durante M horas, e, no dia seguinte, 1 pessoa trabalhou por M horas.

Entao, como o trabalho total realizado no campo grande deve ser o dobro do 
trabalho realizado no campo pequeno, temos que:
N * M + N/2 * M = 2 * ( N/2 * M + 1 * M )
de onde N=4

Portanto, havia 4 trabalhadores no grupo.

[]'s
Rogerio Ponce
---
   
Um  grupo de trabalhadores tinha a tarefa de realizar a colheita de 
dois  campos de trigo, um dos quais tinha o dobro da área do outro. 
Durante meio dia todo o pessoal do grupo trabalhou no campo de trigo grande.

Depois do almoço, metade do pessoal continuou no campo de trigo grande e a  
outra metade trabalhou no campo de trigo pequeno. Durante a tarde acabaram 
ambos as tarefas exceto uma pequena área  do campo  de trigo menor, em cuja 
colheita se ocupou todo o dia seguinte apenas um  trabalhador. 
Quantos trabalhadores haviam neste grupo?

Claudio.



   Flickr agora em português. Você clica, todo mundo vê. Saiba mais.

Re: [obm-l] Problema

2007-11-20 Thread saulo nilson
2 -35 - 48
2 -5-  -3-  x
1 -1 - -9- 45
T*(k1P+k2A)=N
3*(2k1+k2*5)=48
9(k1+k2)=45
2k1+5k2=16
2k1+2k2=10
k2=2
k1=3
N=5(6+6)=60


On 11/16/07, Antonio Manuel Castro del Rio <[EMAIL PROTECTED]>
wrote:
>
> Alguém teria a solução do seguinte problema.
>
> Dois profissionais e cinco aprendizes, produzem 48 peças em 3 dias; um
> profissional e um aprendiz produzem 45 peças em 9 dias. Quantas peças são
> produzidas por 2 profissionais e 3 aprendizes em 5 dias? Resposta 60 peças.
>
>
>


[obm-l] Problema Combinatória

2008-03-26 Thread MauZ
Olá a todos!

Numa estante com 24 livros, de quantas maneiras posso retirar 5 livros sem
ter nenhum consecutivo? E no caso de n livros, quantas maneiras retiro p
livros sem ter nenhum consecutivo?



Pra completar vou colocar parte da minha tentativa de solução, preciso de
ajuda pra saber se está certo até onde fiz e como finalizar pois empaquei.

Fiz dessa forma: Todas Combinações - Combinações c/ Consecutivos

Todas: 24!/5!19!
Consecutivos: 23!/4!19! + 22!/3!19! + 21!/2!19! + 20!/1!19!

Fiz uma formula geral com n e p e deu o seguinte:

n!/p!(n-p!) - [(n-1)!/(p-1)!(n-p)! +
(n-2)!/(p-2)!(n-p)!+...+(n-p+1)!/(n-p)!]

Fatorando deu:

(1/(n-p)!)[n!/p!-(n-1)!/(p-1)!-(n-2)!/(p-2)-...-(n-p+1)!/(n-p)!]

Dae empaquei de vez... Não consegui continuar!
Quem souber fazer por favor me dê a luz! Ou simplesmente indique o erro no
meu raciocínio.

Agradeço antecipadamente,
Maurizio


[obm-l] problema análise

2009-01-07 Thread Murilo Krell
Prezados amigos,poderiam me ajudar com esses problemas?

a) Se uma sequência é monótona tem uma subsequência convergente, prove que a
sequência é, ela própria convergente.

b) A fim de que o número real a seja valor de aderência de (xn) é necessário
e suficiente, que, para todo eps>0 e todo k pertencente a N dados, exista
n>k tal que o modulo de xn-a

Re: [obm-l] Problema!!

2009-04-13 Thread Paulo Santa Rita
Ola Antonio e demais
colegas desta lista ... OBM-L,
(escreverei sem acentos)

E bem conhecido que " um numero natural  pode ser escrito como soma de
dois quadrados se, e somente se, na sua decomposicao em fatores primos
os fatores da forma 4N+3 tenham expoente par ". Como 96=(2^5)*3, ve-se
que o fator primo 3 ( que e da forma 4N+3 ) nao tem expoente par.
Logo, o numero 96 nao pode ser representado como soma de dois
quadrados.

Este tema da representacao de numeros como soma de dois quadrados e
bem conhecido e, em geral,  abordado nos cursos iniciais sobre teoria
dos numeros.

um abraco a todos
PSR, 21304091042




Um n

2009/3/24 Antonio Manuel Castro del Rio :
> Ola, boa noite.
> Preciso de ajuda para resolver um problema.
>
>   COMO FAZER 96 VIRAR UMA SOMA DE DOIS QUADRADOS?
>
> Desde já, obrigado
>    Antonio del Rio
>
>

=
Instru��es para entrar na lista, sair da lista e usar a lista em
http://www.mat.puc-rio.br/~obmlistas/obm-l.html
=


[obm-l] problema interessante!!!

2009-04-29 Thread Cleuber Eduardo
Bom, amigos da lista estou pensando nesse problema a alguns dias, no entanto a 
forma como o fiz é bastante enfadonha.1.Let 
in the exterior the equilateral triangle ABC be a right triangle (∠A = 90◦). On 
the hypotenuse BC constructBCD. Prove that the lengths of the segments AB,AC, 
and AD cannot all be rational.donha. Obrigado desde já.


  Veja quais são os assuntos do momento no Yahoo! +Buscados
http://br.maisbuscados.yahoo.com

Re: [obm-l] Problema

2002-12-17 Thread Cláudio \(Prática\)
8 escrivaninhas certamente são suficientes.

Se cada professor usar uma escrivaninha 90% do tempo, então o número total
de "escrivaninhas-tempo" utilizadas será igual a 8 * 0,90 = 7,2. Assim,
supondo que não exista 0,2 escrivaninha, 8 escrivaninhas são também
necessárias.

Um abraço,
Claudio Buffara.

- Original Message -
From: "Tertuliano Carneiro de Souza Neto" <[EMAIL PROTECTED]>
To: <[EMAIL PROTECTED]>
Sent: Tuesday, December 17, 2002 2:48 PM
Subject: [obm-l] Problema


Alguém pode me ajudar no seguinte problema?



O departamento de matemática tem 8 professores
graduados ocupando o mesmo gabinete. Cada um tanto
estuda em casa como no gabinete. Quantas escrivaninhas
deve haver no gabinete de modo que cada um tenha uma
pelo menos 90% do tempo?

Grato.
Tertuliano Carneiro,
De Salvador.

___
Yahoo! GeoCities
Tudo para criar o seu site: ferramentas fáceis de usar, espaço de sobra e
acessórios.
http://br.geocities.yahoo.com/
=
Instruções para entrar na lista, sair da lista e usar a lista em
http://www.mat.puc-rio.br/~nicolau/olimp/obm-l.html
O administrador desta lista é <[EMAIL PROTECTED]>
=

=
Instruções para entrar na lista, sair da lista e usar a lista em
http://www.mat.puc-rio.br/~nicolau/olimp/obm-l.html
O administrador desta lista é <[EMAIL PROTECTED]>
=



[obm-l] Problema Antigo

2002-12-27 Thread Helder Oliveira de Castro
Oi turma, tubo bem? 

Foi proposto um problema na lista com o seguinte enunciado: 

Num paralelogramo ABCD,uma reta passando por C intercepta a digonal BD em F 
e o lado AB em E. Calcular BE = x, em função de AB = a, sabendo que a área 
do quadrilátero AEFD é o triplo da área do triangulo BCF. 

Eu tentei resolver e encontrei uma solução bastante interessante e 
não-trabalhosa. Para isto segue-se as notações: 

BE = x (por hipótese) 
AB = CD = a (por hipótese) 
h: altura do paralelogramo 
h': altura do triângulo FBE relativa ao lado BE 

Vamos começar. Pelo enunciado temos (AEFD) = 3(BCF) (1). Pelas notações: 

(AEFD) = (ABD) - (FBE) = (ah)/2 - (xh')/2 (2) 
(BCF) = (BEC) - (FBE) = (xh)/2 - (xh')/2  (3) 

Vejamos que os triângulos FBE e FCD são semelhantes pelo caso AA. Assim 
temos que 

x/h' = a/(h-h') (4) 

De (4) vem que h' = (hx)/(a+x). Troque (2) e (3) em (1), simplifique os 
termos semelhantes e depois troque h'. Simplificando novamente e eliminando 
o denominador ficamos com a equação 

x^2 + 2ax - a^2 = 0 

Resolvendo vem x = a(raiz{2}-1). 

Valeus, 
   Helder. 

_
Voce quer um iGMail protegido contra vírus e spams? 
Clique aqui: http://www.igmailseguro.ig.com.br

=
Instruções para entrar na lista, sair da lista e usar a lista em
http://www.mat.puc-rio.br/~nicolau/olimp/obm-l.html
O administrador desta lista é <[EMAIL PROTECTED]>
=



[obm-l] Problema "t"

2003-01-10 Thread Bruno



Olá pessoal,
Eu estava tentando este problema e não conseguiu.
"cos(p/65).cos(2p/65).cos(4p/65).cos(8p/65).cos(16p/65).cos(32p/65) é igual a:
a)1/2
b)1/8
c)1/32
d)1/64
e)1  "
Até


Re: [obm-l] Problema

2003-01-13 Thread Rodrigo Villard Milet
Se você sabe um pouco de álgebra linear fica fácil. Seja k o menor natural
tal que A^k = 0. Portanto, existe um vetor v de R^n tal que A^(k-1) * v não
é zero. Agora provamos que v, Av, ... , A^(k-1) * v são um conjunto l.i.
Suponha que temos a(0)*v + a(1)*Av + ... + a(k-1)*A^(k-1) * v = 0, com a(i)
reais. Multiplique essa equação por A^(k-1) à esquerda, daí segue que
a(0)=0. Depois multiplique por A^(k-2) e terá que a(1)=0. Dessa mesma forma,
mostramos que a(0)=a(1)=...=a(k-1)=0, logo o conjunto é l.i. Isso prova em
particular que k<=n, pois não podemos ter mais de n vetores l.i em R^n.
Voltando ao seu problema se temos que A^(n+1)=0 é pq n+1 não pode ser o
menor número k tal que A^k=0 (pelo OBS acima). Então k<=n. Se k=n, acabou,
se k é menor que n, segue que A^n = A^k * A^(n-k) = 0.
Talvez tenha um jeito mais simples pra fazer isso..
Abraços,
 Villard

-Mensagem original-
De: [EMAIL PROTECTED] <[EMAIL PROTECTED]>
Para: [EMAIL PROTECTED] <[EMAIL PROTECTED]>
Data: Segunda-feira, 13 de Janeiro de 2003 18:34
Assunto: [obm-l] Problema


>
>OLa galera,
>
>Estou enviando um bom problema de matriz. La vai...
>Seja  A uma matriz nxn. Prove que se A^(n+1) = 0, então A^n = 0.
>
>Cícero Thiago
>
>
>
>
>
>--
>Use o melhor sistema de busca da Internet
>Radar UOL - http://www.radaruol.com.br
>
>
>
>=
>Instruções para entrar na lista, sair da lista e usar a lista em
>http://www.mat.puc-rio.br/~nicolau/olimp/obm-l.html
>O administrador desta lista é <[EMAIL PROTECTED]>
>=
>

=
Instruções para entrar na lista, sair da lista e usar a lista em
http://www.mat.puc-rio.br/~nicolau/olimp/obm-l.html
O administrador desta lista é <[EMAIL PROTECTED]>
=



Re: [obm-l] Problema

2003-01-17 Thread bene



 

  Corrigindo,a alternativa  (b) é 400
  

  

  Nos festejos juninos, 20 casais de dançarinos são 
  colocados em círculo de tal maneira que um homem e uma mulher formando 
  um par estão situados diametralmente opostos. Durante a dança, dois 
  dançarinos adjacentes trocam de lugar enquanto todos os outros 
  permanecem na mesma posição. Essa mudança é repetida com pares 
  adjacentes até que, na posição final, os dois dançarinos de cada par 
  estejam novamente diametralmente opostos, mas na posição contrária da 
  inicial. Então o número mínimo de mudanças, de dois dançarinos 
  adjacentes, para acontecer isso é:
  (a) 20! (b) 400 (c) 10! (d) 19! (e) 
  20


[obm-l] problema antigo!

2003-01-17 Thread Erasmo de Souza Dias
Olá pessoas! 
Preciso de ajuda para resolver tal problema:
Determine todos os pares (x,y) de inteiros positivos tal que x^(x+y)=y^(y-x).
Agradeço a colaboração... um forte abraço.Busca Yahoo! 
O melhor lugar para encontrar tudo o que você procura na Internet

Re: [obm-l] problema

2003-02-07 Thread Augusto Cesar de Oliveira Morgado
1) Por favor, quilometro se abrevia km, com k minusculo.
2) 1 no = 1,852 km/h
10 nos = 18,52 km/h
A resposta eh 370,4/18,52= 20 horas


Em Fri, 7 Feb 2003 08:19:37 -0300 (ART), elton francisco ferreira 
<[EMAIL PROTECTED]> disse:

> Se o nó é igual a uma milha marítima por hora e uma
> milha marítima é igual a 1852 m; quanto tempo uma
> embarcação com velocidade constante de 10 nós gasta
> para percorrer 370,4 Km?
> 
> ___
> Busca Yahoo!
> O melhor lugar para encontrar tudo o que você procura na Internet
> http://br.busca.yahoo.com/
> =
> Instruções para entrar na lista, sair da lista e usar a lista em
> http://www.mat.puc-rio.br/~nicolau/olimp/obm-l.html
> O administrador desta lista é <[EMAIL PROTECTED]>
> =
> 
> 

=
Instruções para entrar na lista, sair da lista e usar a lista em
http://www.mat.puc-rio.br/~nicolau/olimp/obm-l.html
O administrador desta lista é <[EMAIL PROTECTED]>
=



Re: [obm-l] problema

2003-02-07 Thread Tertuliano Carneiro
Olá!

Do enunciado, um nó equivale a 1,852 km/h. Isso é
tudo.



 --- elton francisco ferreira
<[EMAIL PROTECTED]> escreveu: > Se o nó é
igual a uma milha marítima por hora e uma
> milha marítima é igual a 1852 m; quanto tempo uma
> embarcação com velocidade constante de 10 nós gasta
> para percorrer 370,4 Km?
> 
>
___
> Busca Yahoo!
> O melhor lugar para encontrar tudo o que você
> procura na Internet
> http://br.busca.yahoo.com/
>
=
> Instruções para entrar na lista, sair da lista e
> usar a lista em
> http://www.mat.puc-rio.br/~nicolau/olimp/obm-l.html
> O administrador desta lista é
> <[EMAIL PROTECTED]>
>
= 

___
Yahoo! GeoCities
Tudo para criar o seu site: ferramentas fáceis de usar, espaço de sobra e acessórios.
http://br.geocities.yahoo.com/
=
Instruções para entrar na lista, sair da lista e usar a lista em
http://www.mat.puc-rio.br/~nicolau/olimp/obm-l.html
O administrador desta lista é <[EMAIL PROTECTED]>
=



Re: [obm-l] problema

2003-02-07 Thread Leahpar Xarm
370,4Km=370400m=200*1852=200* "nós"
dividindo o percurso pela velocidade tem-se o tempo: 20h
 elton francisco ferreira <[EMAIL PROTECTED]> wrote:
Se o nó é igual a uma milha marítima por hora e umamilha marítima é igual a 1852 m; quanto tempo umaembarcação com velocidade constante de 10 nós gastapara percorrer 370,4 Km?___Busca Yahoo!O melhor lugar para encontrar tudo o que você procura na Internethttp://br.busca.yahoo.com/=Instruções para entrar na lista, sair da lista e usar a lista emhttp://www.mat.puc-rio.br/~nicolau/olimp/obm-l.htmlO administrador desta lista é <[EMAIL PROTECTED]>=Busca Yahoo! 
O serviço de busca mais completo da Internet. O que você pensar o Yahoo! encontra.

[obm-l] problema selecionados

2003-02-07 Thread Daniel Pini



Olá queria muito saber se alguém aqui desta lista 
conseguiu resolver todos os exercícios do livro Problemas Selecionados de 
Matemática. Porque eu realmente os acho muito difíceis e trabalhosos. 
Daniel.


[obm-l] Problema 01

2003-02-07 Thread elton francisco ferreira
A circunferencia de um relógio mede 37,68. Qual é a
mdida do menor arco formado pelos ponteiros de um
relógio as 4h? e do arco maior?

___
Busca Yahoo!
O melhor lugar para encontrar tudo o que você procura na Internet
http://br.busca.yahoo.com/
=
Instruções para entrar na lista, sair da lista e usar a lista em
http://www.mat.puc-rio.br/~nicolau/olimp/obm-l.html
O administrador desta lista é <[EMAIL PROTECTED]>
=



[obm-l] Problema 02

2003-02-07 Thread elton francisco ferreira
Um casal de namorados foi a um parque de diversões. A
roda-gigante tem 10 cadeiras e 8 m de raio. O garoto,
que foi o primeiro a entrar, sentou-se na cadeira
número 1. A garota sentou-se na cadeira número 7 e
desceu depois de dar 20 voltas completas. Quantos
metros o garoto percorreu, do instante em que subiu no
brinquedo até o momento em que a garota desceu? 
pi: 3,14

___
Busca Yahoo!
O melhor lugar para encontrar tudo o que você procura na Internet
http://br.busca.yahoo.com/
=
Instruções para entrar na lista, sair da lista e usar a lista em
http://www.mat.puc-rio.br/~nicolau/olimp/obm-l.html
O administrador desta lista é <[EMAIL PROTECTED]>
=



[obm-l] Problema 03

2003-02-09 Thread elton francisco ferreira
Deseja-se construir um anel rodoviário circular em
orno da cidade de São Paulo, distando aproximadamente
20km da praça da sé. quantos km terá essa rodovia e
qual a densidade demográfica da região interior ao
anel, supondo que lá residam 12 milhões de pessoasa?
pi:3,14 

___
Busca Yahoo!
O melhor lugar para encontrar tudo o que você procura na Internet
http://br.busca.yahoo.com/
=
Instruções para entrar na lista, sair da lista e usar a lista em
http://www.mat.puc-rio.br/~nicolau/olimp/obm-l.html
O administrador desta lista é <[EMAIL PROTECTED]>
=



[obm-l] Problema 04

2003-02-10 Thread elton francisco ferreira
Um caixa automática de um banco só libera notas de R$
5,00 e R$ 10,00. Uma pessoa retirou dessa caixa a
importância de R$ 65,00, recebendo 10 notas. O produto
do número de notas de R$ 5,00 pelo número de notas de
R$ 10,00 é igual a

16 
25
24
21 


___
Busca Yahoo!
O melhor lugar para encontrar tudo o que você procura na Internet
http://br.busca.yahoo.com/
=
Instruções para entrar na lista, sair da lista e usar a lista em
http://www.mat.puc-rio.br/~nicolau/olimp/obm-l.html
O administrador desta lista é <[EMAIL PROTECTED]>
=



[obm-l] Problema 05

2003-02-11 Thread elton francisco ferreira
Numa prova de matemática, um aluno deve responder a 60
itens do tipo verdadeiro ou falso. para cada item
respondido corretamente, o aluno vai ganhar 2 pontos
e, para cada item que errar, vai perder 1 ponto. A
nota do aluno é função do número de itens que ele
acertar. Se o aluno obteve 30 pontos, quantos itens
ele acertou?

 

___
Busca Yahoo!
O serviço de busca mais completo da Internet. O que você pensar o Yahoo! encontra.
http://br.busca.yahoo.com/
=
Instruções para entrar na lista, sair da lista e usar a lista em
http://www.mat.puc-rio.br/~nicolau/olimp/obm-l.html
O administrador desta lista é <[EMAIL PROTECTED]>
=



[obm-l] Problema interessante

2003-02-24 Thread Cláudio \(Prática\)
Title: Help



Taí um resultado inesperado (pelo menos pra mim):
 
Tome uma partição QUALQUER de {1,2,...,2n} em dois conjuntos A e B com n 
elementos cada.  Ponha os elementos de A em ordem 
crescente a_1<..>b_n. 
 Prove que:|a_1-b_1| + ... + |a_n-b_n| = n^2.
 
Um abraço,
Claudio.


Re: [obm-l] problema

2003-02-24 Thread Cláudio \(Prática\)


> Dos 800 sargentos formados pela EsSa a cada ano, 5%
> pedem para sair do exército ao completarem 5 anos de
> serviço, a quantidade de sargentos formados pela EsSa
> após 12 anos e que estão em atividade é?
>

Supondo que no 5o. aniversário de formatura de uma dada turma, os 5% saem e
os 760 restantes ficam até a aposentadoria (que não ocorre antes de 13
anos), então a solução é:

800 recém formados
800 formados há 1 ano
...
800 formados há 4 anos  ==> Sub-Total = 5*800 = 4.000

760 formados há 5 anos
...
760 formados há 12 anos  ==> Sub-Total = 8*760 = 6.080

Total em atividade = 10.080.

Um abraço,
Claudio.

=
Instruções para entrar na lista, sair da lista e usar a lista em
http://www.mat.puc-rio.br/~nicolau/olimp/obm-l.html
O administrador desta lista é <[EMAIL PROTECTED]>
=


[obm-l] problema limite

2003-02-28 Thread Bruno



Olá a todos,
Qualquer ajuda, eu agradeço:
"Calcule:
lim { sqrt[x+sqrt(x)] - sqrt(x-1) }  = ?
x-->mais infinito
"
Até...
Bruno


[obm-l] Problema simples...

2003-03-04 Thread leandro








Amigos,

 

Esse e um problema simples, mas eu nao consegui ver a
solucao da parte c:

 

 

Seja u,v vetores em R^n e A=uvT.  Entao,
mostre que

 

 

(a)  
A^2 = (u.v) A.  Esse eu fiz.
(u.v denota o produto interno)

(b)  
Use a parte (a) para mostrar que se
u.v e diferente de zero, entao (u.v) e o unico autovalor diferente de zero de
A.  (Esse eu fiz)

(c)  
Use a parte (a) e a parte (b) para
mostrar que se A tem posto 1, entao I-A e inversivel se e somente se A^2 e
diferente de A. 

 

 

So nao consegui ver a parte (c). Alguem pode me
ajudar. 

 

 

Leandro. 








  1   2   3   4   5   6   7   8   9   10   >